La contracción del campo de fermiones en QED sin masa 1 + 1-dimensional

Mi pregunta proviene del libro de texto de Peskin & Schroeder, la integral (19.26):

d 2 k ( 2 π ) 2 mi i k ( y z ) i k k 2 = ∂̸ ( i 4 π registro ( y z ) 2 )

Pregunta: ¿cómo derivar la fórmula del lado izquierdo al lado derecho?

Si considerando la identidad (3.117) y el conjunto metro = 0 , Tengo

d 2 k ( 2 π ) 2 i k γ k 2 mi i k ( y z ) = i ∂̸ ( D R ( y z ) )
aquí
D R ( y z ) = d 2 k ( 2 π ) 2 i k 2 mi i k ( y z )
el 2-vector: k m = ( k 0 , k 1 ) y debido a la condición sin masa: ( k 0 ) 2 = ( k 1 ) 2 . colocar k k 1 .por lo tanto tengo
+ d k 1 ( 2 π ) [ 1 2 k 0 mi i [ k 0 ( y z ) 0 k 1 ( y z ) 1 ] + 1 2 k 0 mi i [ k 0 ( y z ) 0 k 1 ( y z ) 1 ] ] = i 4 π   2 + pecado ( k ( y z ) 0 ) k mi i k ( y z ) 1 d k

Pero no pude obtener el término logarítmico de la fórmula anterior.

NOTA Encontré una respuesta relacionada Una integral de cuatro dimensiones en Peskin & Schroeder

Uno no puede usar la condición en el caparazón aquí, ya que los componentes de k m son independientes

Respuestas (1)

Encontré que una transformada de Fourier puede responder esta pregunta. Permítanme trabajar en el espacio euclidiano por la rotación de Wick: k 0 i k 2 , por eso

d 2 k ( 2 π ) 2 i k k 2 mi i k ( y z ) d k 1 d k 2 ( 2 π ) 2 i k | k | 2 mi i k ( y z ) colocar: y z := r = i r 1 ( 2 π ) 2 1 k 1 2 + k 2 2 mi i ( k 1 r 1 + k 2 r 2 ) d k 1 d k 2 = i r ( 1 ( 2 π ) 2 [ 2 π registro | r | ] ) = i r ( 1 4 π registro ( y z ) 2 )
aquí la parte azul es una integral de Fourier en 2-D, esta transformada de Fourier está presente en la función de Green del operador de Laplace en un espacio bidimensional.

i a la cabeza de RHS de la primera línea se cae. Y registro | r | = registro | y z | 2 1 / 2 = registro ( ( y z ) 2 ) 1 / 2 . Así que la última línea es
i r ( 1 4 π registro ( ( y z ) 2 ) ) .
Entonces está de acuerdo con otro método . La segunda línea de (19.26) en el libro de texto está mal impresa.